JEE Main & Advanced Mathematics Binomial Theorem and Mathematical Induction Question Bank Critical Thinking

  • question_answer
    If the coefficient of x in the expansion of \[{{\left( {{x}^{2}}+\frac{k}{x} \right)}^{5}}\] is 270, then k = [EAMCET 2002]

    A) 1

    B) 2

    C) 3

    D) 4

    Correct Answer: C

    Solution :

    \[{{T}_{r+1}}={}^{5}{{C}_{r}}{{({{x}^{2}})}^{5-r}}{{\left( \frac{k}{x} \right)}^{r}}\] For coefficient of x, \[10-2r-r=1\,\,\,\,\Rightarrow r=3\] Hence, \[{{T}_{3+1}}={}^{5}{{C}_{3}}{{({{x}^{2}})}^{5-3}}{{\left( \frac{k}{x} \right)}^{3}}\] According to question, \[10\,{{k}^{3}}=270\,\,\Rightarrow \,\,k=3\].


You need to login to perform this action.
You will be redirected in 3 sec spinner